I have to calculate $\lim_{x \to \infty}{x-x^2\ln(1+\frac{1}{x})}$. I rewrote it as $\lim_{x \to \infty}{\frac{x-x^3\ln^2(1+\frac{1}{x})}{1 + x\ln(1+\frac{1} {x})}}$ and tried to apply L'Hôpital's rule but it didn't work. How to end this?
-
1You could try substituting $\frac{1}{x}=t$ – Arpan Mar 26 '15 at 08:40
-
4See also http://math.stackexchange.com/questions/1702282/using-lhospital-solve-lim-x-to-infty-x-x2-ln1-frac1x – Martin Sleziak Nov 09 '16 at 06:27
2 Answers
Make the substitution $t=\dfrac{1}{x}$. Then, $x\to\infty \implies t\to 0$
$$\lim_{x\to\infty}\left(x-x^2\ln\left(1+\frac{1}{x}\right)\right)\\ = \lim_{t\to 0}\left(\frac{1}{t}-\frac{1}{t^2}\ln(1+t)\right)\\= \lim_{t\to 0}\left(\frac{t-\ln(1+t)}{t^2}\right)$$
This comes out as $\frac{0}{0}$ on direct plugging of values, so it's ready for some L'Hopital bash.
$$\lim_{t\to 0}\left(\dfrac{1-\frac{1}{1+t}}{2t}\right)$$
Again, by direct plugging, we get $\frac{0}{0}$, so recursion of L'Hopital will work.
$$\lim_{t\to 0}\left(\dfrac{0+\frac{1}{(1+t)^2}}{2}\right)=\lim_{t\to 0}\left(\frac{1}{2(1+t)^2}\right)=\boxed{\frac{1}{2}}$$

- 6,334
-
Ideally, you should simplify your limit $ \displaystyle \lim_{t \to 0} \left ( \frac {1 - \frac 1{1+t}} {2t} \right ) $ first. – GohP.iHan Apr 21 '15 at 04:36
-
@GohP.iHan, well, I was thinking that doing so will make it messier, so I continued from that. Simpler way for the win! :D – Prasun Biswas Apr 21 '15 at 05:30
The reason why L'Hôpital's rule didn't work is that you added on a determined-term into the limit, i.e. $$ 1 + x \ln \left( 1 + \frac{1}{x} \right) = 1 + \ln \left( \left( 1 + \frac{1}{x} \right)^x \right) \to 1 + \ln \left( e \right)= 2 $$ when $x \to \infty$. Another approach with the one from Prasun Biswas is using the Taylor series for $1/x$, for $x>1$: \begin{align} x - x^2 \ln \left( 1 + \frac{1}{x} \right) &= x - x^2 \left( \sum_{k=1}^{\infty} \frac{(-1)^{k+1}}{ k} \frac{1}{x^k} \right) = x - x^2 \left( \frac{1}{x} - \frac{1}{2x^2} + \sum_{k=3}^{\infty} \frac{(-1)^{k+1}}{k} \frac{1}{x^k} \right) \\ &= \frac{1}{2} + \sum_{k=3}^{\infty} \frac{(-1)^{k+1}}{k} \frac{1}{x^{k-2}} \end{align} Letting $x \to \infty$, one ends up the limit with $\frac{1}{2}$.

- 81